LSAT and Law School Admissions Forum

Get expert LSAT preparation and law school admissions advice from PowerScore Test Preparation.

 voodoochild
  • Posts: 185
  • Joined: Apr 25, 2012
|
#9255
Experts,
I was trying to diagram these two sentences from Pt24, S2 Q21

I believe there is a difference between
"Ineffective law is unenforceable."
and
"No effective law is unenforceable"
because the first can be diagrammed as Enforceable :arrow: Effective.

The second one as Effectuve :arrow: Enforceable

Am I correct?
User avatar
 Dave Killoran
PowerScore Staff
  • PowerScore Staff
  • Posts: 5853
  • Joined: Mar 25, 2011
|
#9267
Hi Voodoo,

Yes, correct. I will note for others reading that you've presented the contrapositive of the first statement. Read as-is, I would normally diagram that as Effective :arrow: Enforceable, but the contrapositive removes both negatives, which is helpful.

Thanks!
 GLMDYP
  • Posts: 100
  • Joined: Aug 19, 2013
|
#10391
Hi Powerscore!
I chose the answer (C), but the answer is (A). Can you please explain the difference?
Thanks!
 Steve Stein
PowerScore Staff
  • PowerScore Staff
  • Posts: 1153
  • Joined: Apr 11, 2011
|
#10536
Hi GLMDYP,

Thanks for your question. It would be helpful to know how you broke that stimulus down; how did you approach the question?

Please let us know--thanks!

~Steve
 pacer
  • Posts: 57
  • Joined: Oct 20, 2014
|
#17479
I am really confused with this question

I diagrammed the conditional statement from the stimulus as

not effective -> not enforceable

and its contrapositive

enforceable -> effective

Then I diagrammed the answer choices

A) effective -> enforceable, not enforceable -> not effective

B) enforceable -> effective, not effective -> not enforceable



Answer choice B diagram matches my diagram for the stimulus

But the correct answer is A. I am really confused

This question is asking for an assumption that is sufficient to lead from the premise to the conclusion.

The above is an explanation of how I attempted to go through the problem. I am not sure where I have made the mistake and why my method does not lead to the correct answer. Can you please explain this question and help point out where I am going wrong with my approach.

Thanks,

Pacer
 Elizabeth Mulkey
PowerScore Staff
  • PowerScore Staff
  • Posts: 10
  • Joined: Oct 02, 2012
|
#17492
Hi Pacer,

Good job catching the conditional reasoning in this Justify the Conclusion question. Here, we're looking for an assumption that connects the ideas of "enforceability" and "effectiveness" - answer choices (A) and (B) both do that, so you were correct in narrowing it down to these two.

We can diagram the conditional statement "Ethical qualms notwithstanding, when a law fails to be effective, it should not be a law" as not effective :arrow: not law. Its contrapositive would be law :arrow: effective. Though this looks like answer choice (B), this doesn't tell us anything about enforceable laws.

Here's what we get in the question stem: gambling laws are unenforceable; when a law fails to be effective, it should not exist; gambling laws should not exist. Answer choice (A) connects "unenforceable" with "not effective." Your diagram of the answer choices looks good, but remember that we're dealing with distinct ideas in the editor's argument, and the correct answer here is the one that connects them. Hope this helps!

- Elizabeth
 swt2003
  • Posts: 5
  • Joined: Apr 28, 2017
|
#36857
A. No effective law is unenforceable - This answer choice can be translated to All effective laws are enforceable. No X's are Y's is the same as All X's are NOT Y's. In this case, we are negating "unenforceable" which becomes enforceable. Then reverse and negate to get the contrapositive.

- Enforceable ----> - Effective

We can feed this into our conditional statement in the passage:

- Effective ----> - Law

we combine the two statements

By combining, we see that if a law is not enforceable then it's not effective, we know from the passage if a law is not effective it should not be a law.

Not Enforceable ----> Not Effective ------> Not Law
 Adam Tyson
PowerScore Staff
  • PowerScore Staff
  • Posts: 5153
  • Joined: Apr 14, 2011
|
#37169
Exactly right, swt! Well done!
 Oneshot06
  • Posts: 21
  • Joined: Feb 07, 2018
|
#43927
Hi, I'm having a bit of a tough time reasoning with this question. Could you please verify my thought process below and tell me where I'm going wrong?

This is a Justify Conclusion question so the answer has to be a sufficient assumption that comes to a valid conclusion.

Conclusion: There should be no legal prohibition against gambling.
Premise: regardless of effort, laws are unenforceable.
Premise: law -effective ---> -law, or contrapositive, law---->effective, but it also has to be enforceable.

The part that's missing is linking the gap between law, effective and enforceable. So if there should be "no legal prohibition against gambling" it's because the law is ineffective and unenforceable.

This is where I get stuck in my thought process and am a bit confused by the wording of answer choice A.) No effective law is unenforceable. So worded differently, it would mean if the law is effective, then it's enforceable? This further means it would justify the conclusion of there not being a legal prohibition for gambling, since it's neither effective or enforceable? Also if answer B.) was worded "All effective laws are enforceable," would it then be the correct answer?

Thanks!
 Adam Tyson
PowerScore Staff
  • PowerScore Staff
  • Posts: 5153
  • Joined: Apr 14, 2011
|
#43935
You're pretty close there, Oneshot! However, the concept of laws against gambling being unenforceable is already in the stimulus - see the end of the first sentence. That part is not the missing link, and our answer doesn't need to tell us that since we already have it in a premise. Instead, the gap is just between effectiveness and enforceability. Answer A gives us that connection, and your interpretation of it is accurate.

You're correct about what would happen if we were to change answer B to your alternative - that would make it logically identical in meaning to answer A, with effectiveness being sufficient for enforceability, closing our gap and justifying the conclusion.

Here's my breakdown of the logic at work here:

Premise: laws should be effective
Premise: gambling prohibitions are not enforceable
Conclusion: Gambling prohibitions should not be laws

The missing link here is "prohibitions that are not enforceable are not effective".

Good work!

Get the most out of your LSAT Prep Plus subscription.

Analyze and track your performance with our Testing and Analytics Package.